Difference between revisions of "2019 AMC 10A Problems/Problem 9"

(Solution 1)
m (Solution)
Line 5: Line 5:
 
<math>\textbf{(A) } 995 \qquad\textbf{(B) } 996 \qquad\textbf{(C) } 997 \qquad\textbf{(D) } 998 \qquad\textbf{(E) } 999</math>
 
<math>\textbf{(A) } 995 \qquad\textbf{(B) } 996 \qquad\textbf{(C) } 997 \qquad\textbf{(D) } 998 \qquad\textbf{(E) } 999</math>
  
==Solution==
+
==Solutions==
  
 
===Solution 1===
 
===Solution 1===
Line 15: Line 15:
  
 
Following from the fact that <math>n+1</math> must be prime, we can use to answer choices as possible solutions for <math>n</math>. <math>A</math>, <math>C</math>, and <math>E</math> don't work because <math>n+1</math> is even, and <math>D</math> does not work since <math>999</math> is divisible by <math>9</math>. Thus, the only correct answer is <math>996 \implies \boxed{\textbf{(B)}}</math>.
 
Following from the fact that <math>n+1</math> must be prime, we can use to answer choices as possible solutions for <math>n</math>. <math>A</math>, <math>C</math>, and <math>E</math> don't work because <math>n+1</math> is even, and <math>D</math> does not work since <math>999</math> is divisible by <math>9</math>. Thus, the only correct answer is <math>996 \implies \boxed{\textbf{(B)}}</math>.
 +
 +
===Solution 3===
 +
There are only 5 numbers to test, so just test them to get B.
  
 
==See Also==
 
==See Also==

Revision as of 23:33, 11 February 2019

Problem

What is the greatest three-digit positive integer $n$ for which the sum of the first $n$ positive integers is $\underline{not}$ a divisor of the product of the first $n$ positive integers?

$\textbf{(A) } 995 \qquad\textbf{(B) } 996 \qquad\textbf{(C) } 997 \qquad\textbf{(D) } 998 \qquad\textbf{(E) } 999$

Solutions

Solution 1

Because the sum of $n$ positive integers is $\frac{(n)(n+1)}{2}$, and we want this to not be a divisor of the $n!$, $n+1$ must be prime. The greatest three-digit integer that is prime is $997$. Subtract $1$ to get $996 \implies \boxed{\textbf{(B)}}.$

-Lcz

Solution 2

Following from the fact that $n+1$ must be prime, we can use to answer choices as possible solutions for $n$. $A$, $C$, and $E$ don't work because $n+1$ is even, and $D$ does not work since $999$ is divisible by $9$. Thus, the only correct answer is $996 \implies \boxed{\textbf{(B)}}$.

Solution 3

There are only 5 numbers to test, so just test them to get B.

See Also

2019 AMC 10A (ProblemsAnswer KeyResources)
Preceded by
Problem 8
Followed by
Problem 10
1 2 3 4 5 6 7 8 9 10 11 12 13 14 15 16 17 18 19 20 21 22 23 24 25
All AMC 10 Problems and Solutions

The problems on this page are copyrighted by the Mathematical Association of America's American Mathematics Competitions. AMC logo.png